31
Get Prepped! PrepTest 28 1 PrepTest 28 (314-321) 1 1 1 1 1 1 Analysis of the Questions: 1. A student- This is a strengthen or weaken the argument question. Why does the LSAT test your ability to evaluate an argument? Because it is an important skill for law students and lawyers. Strengthening and weakening questions are the second most-commonly asked questions on the LSAT. When dealing with this type of question, ignore issues and ideas that are irrelevant to strengthening or weakening the argument. Do not correct or defend the facts in the passage. To answer these questions correctly, you must understand the authors facts and conclusion. You want to be able to paraphrase the argument and then marshal evidence for or against the authors position. After a bit of practice, finding the key piece of evidence will come naturally. The correct answer here is the choice that fails to strengthen the argument. In other words, it contains either irrelevant information or information that weakens the argument. The author argued that the students past performance would dictate her present performance. Note that this argument is valid only if the past conditions are similar to the present conditions. (A) * This answer is correct because it provides evidence that does not strengthen the argument; instead, it introduces a new condition that weakens the argument. Because the student is now receiving good academic help, her past performance is not as good an indicator of how well she will do this time. (B) This is an incorrect answer; it tends to support the authors argument. Because the student has shown a propensity to earn B grades in a variety of subjects, it is likely that she will earn a B in her present course regardless the subject. (C) This is an incorrect answer; it supports the argument by showing that the student is not studying differently than before. (D) This is an incorrect answer; it tends to strengthen the authors argument by showing that the student has received a B in virtually all of her classes. (E) This is an incorrect answer; it strengthens the authors argument by showing that the present course is similar to a past course. It is safe to assume that similar conditions will yield a similar result. 2. If the government- This is an identify an assumption question. The answer will not be explicitly stated within the passage. To answer the question correctly, you must read between the lines. The passage contains the explicit conclusion that since the overall level of funding would fall, the government should not increase its funding. From the conclusion, work backwards to reveal the hidden assumption, the missing premise, or implicit factual basis. The answer is a logical antecedent, or a necessary pre-condition, of the conclusion. Be leery of an answer choice that merely restates a fact from the passage or an answer choice that introduces a topic not found in the passage. (A) This is an incorrect answer because it contradicts the authors conclusion. The author concluded that, to keep from depressing the overall level of funding, the government should not increase funding for civilian scientific research. This assumption is not a logical step from the facts towards the conclusion. (B) * This answer is correct because it is an unstated assumption upon which the author relied to reach the stated conclusion. An increase in government funding and a decrease in private contributions will lead to an overall decline only if the decrease in private funding is greater than the increase in government funding. (C) Whether researchers whose work receives government funding accept private donations does not allow us to make a conclusion about the overall level of funding. (D) This is an incorrect answer because the conclusion did not depend on this assumption. This answer provides additional information about the relationship between the efficacy of research and sources of funding. It does not address the overall level of funding. (E) This is an incorrect answer because the conclusion did not depend upon this assumption. The current level of funding is not directly relevant to the effect that an increase in government funding would have on overall research dollars. 3. For any given- This is a parallel reasoning question. One way the LSAT tests your ability to evaluate arguments and methods of reasoning is to ask that you mirror an argument. It is important to practice parallel reasoning because it is a difficult and unfamiliar skill. To the uninitiated, these are deadly.

1 1 1 1 1 1 - dl.keywin.orgdl.keywin.org/5/4/54bc444545de55e9a8148ce94764a168.pdf · 1 1 1 1 1 1 Analysis of the ... Why does the LSAT test your ability to evaluate an argument? Because

Embed Size (px)

Citation preview

Page 1: 1 1 1 1 1 1 - dl.keywin.orgdl.keywin.org/5/4/54bc444545de55e9a8148ce94764a168.pdf · 1 1 1 1 1 1 Analysis of the ... Why does the LSAT test your ability to evaluate an argument? Because

Get Prepped! PrepTest 28

1

PrepTest 28 (314-321)

1 1 1 1 1 1 Analysis of the Questions: 1. A student- This is a strengthen or weaken the argument question. Why does the LSAT test your ability to evaluate an argument? Because it is an important skill for law students and lawyers. Strengthening and weakening questions are the second most-commonly asked questions on the LSAT. When dealing with this type of question, ignore issues and ideas that are irrelevant to strengthening or weakening the argument. Do not correct or defend the facts in the passage. To answer these questions correctly, you must understand the author�s facts and conclusion. You want to be able to paraphrase the argument and then marshal evidence for or against the author�s position. After a bit of practice, finding the key piece of evidence will come naturally. The correct answer here is the choice that fails to strengthen the argument. In other words, it contains either irrelevant information or information that weakens the argument. The author argued that the student�s past performance would dictate her present performance. Note that this argument is valid only if the past conditions are similar to the present conditions. (A) * This answer is correct because it provides evidence that does not strengthen the argument; instead, it introduces a new condition that weakens the argument. Because the student is now receiving good academic help, her past performance is not as good an indicator of how well she will do this time. (B) This is an incorrect answer; it tends to support the author�s argument. Because the student has shown a propensity to earn B grades in a variety of subjects, it is likely that she will earn a B in her present course regardless the subject. (C) This is an incorrect answer; it supports the argument by showing that the student is not studying differently than before. (D) This is an incorrect answer; it tends to strengthen the author�s argument by showing that the student has received a B in virtually all of her classes. (E) This is an incorrect answer; it strengthens the author�s argument by showing that the present course is similar to a past course. It is safe to assume that similar conditions will yield a similar result. 2. If the government- This is an identify an assumption question. The answer will not be explicitly stated within the passage. To answer the question correctly, you must read between the lines. The passage contains the explicit conclusion that since the overall level of funding would fall, the government should not increase its funding. From the conclusion, work backwards to reveal the hidden assumption, the missing premise, or implicit factual basis. The answer is a logical antecedent, or a necessary pre-condition, of the conclusion. Be leery of an answer choice that merely restates a fact from the passage or an answer choice that introduces a topic not found in the passage. (A) This is an incorrect answer because it contradicts the author�s conclusion. The author concluded that, to keep from depressing the overall level of funding, the government should not increase funding for civilian scientific research. This assumption is not a logical step from the facts towards the conclusion. (B) * This answer is correct because it is an unstated assumption upon which the author relied to reach the stated conclusion. An increase in government funding and a decrease in private contributions will lead to an overall decline only if the decrease in private funding is greater than the increase in government funding. (C) Whether researchers whose work receives government funding accept private donations does not allow us to make a conclusion about the overall level of funding. (D) This is an incorrect answer because the conclusion did not depend on this assumption. This answer provides additional information about the relationship between the efficacy of research and sources of funding. It does not address the overall level of funding. (E) This is an incorrect answer because the conclusion did not depend upon this assumption. The current level of funding is not directly relevant to the effect that an increase in government funding would have on overall research dollars. 3. For any given- This is a parallel reasoning question. One way the LSAT tests your ability to evaluate arguments and methods of reasoning is to ask that you mirror an argument. It is important to practice parallel reasoning because it is a difficult and unfamiliar skill. To the uninitiated, these are deadly.

Page 2: 1 1 1 1 1 1 - dl.keywin.orgdl.keywin.org/5/4/54bc444545de55e9a8148ce94764a168.pdf · 1 1 1 1 1 1 Analysis of the ... Why does the LSAT test your ability to evaluate an argument? Because

Get Prepped! PrepTest 28

2

Some general notes. These questions usually require you to use symbols to diagram the structure of the argument from the passage. The blueprint, or design, of the argument is your only concern. The correct answer will be presented in the same manner as the argument from the passage. Here we can get away with simply paraphrasing the passage. Do not waste time evaluating the correctness of the argument or its specific content; its structure is the only thing you should care about. Beware of answer choices that deal with the same subject matter or incorporate the same wording as the passage; generally, they are red herrings. The correct answer is not necessarily identical to the passage, but it is the most similar in its reasoning. Now let�s get to work on this passage. First, what is the flaw? It should be pretty obvious. In a 1000 ticket lottery, presumably there are 1, or 2, or 3 tickets that must win. Second, paraphrase the argument. In a closed universe (a 1000 ticket lottery), 999 (or 998 or 997) tickets will lose. Therefore, all 1000 members must lose. Third, eliminate answer choices that mimic the subject of the passage too closely. (B), (C) and (D) all use the number 1000, making it very likely that they are wrong. (A) * This answer is correct; its reasoning most closely follows the pattern of the argument from the passage. To be sure, diagram the argument. If draw 1 card out of 52 its reasonable it will not be an Ace. Therefore, you will never draw an ace. But of course, that is statistically silly, four out of the fifty-two are aces. Both this answer and the argument reason that, because it is unlikely that one out of many (52 or 1000) will be selected, zero will be selected. (B) This is an incorrect answer, it does not follow the same pattern of reasoning. Its diagram may look like this: If 999 out of 1000 = reasonable to win, then 1 out of 1000 = reasonable no win. Note, there is no flaw to this reasoning. Also, it used 1000. (C) This one has it backwards. Its diagram looks like this: If 1000 out of 1000 = not reasonable, then 1000 out of 1000 = never. In this instance, there is a low probability because the outcome depends on the same thing happening every time. (D) This isn�t flawed. There is a reasonable (50%) chance it will be heads. (E) This is an incorrect answer; it does not follow the same pattern of reasoning. This choice follows this pattern of reasoning: If all five-year olds are on average one meter, then this five-year old (Pat) is exactly three feet. This is a flaw, but it is not the correct flaw. 4. Dental Researcher- This is a strengthen the argument question. Identify the piece of evidence that most strongly supports the researcher�s reasoning� that a dentist should not fill cavities unless there is imminent danger. (A) This is an incorrect answer; although it appears to strengthen the argument, it does not. The dental researcher forbade damaging procedures only when there is imminent danger. This answer forbids damaging procedures, but went off the subject by discussing �long term benefits.� (B) This is an incorrect answer; because it is not directly relevant to the reasoning of the passage, it neither strengthens nor weakens the reasoning of the researcher. The passage addressed the costs and benefits of remedial measures; this answer suggests preventive measures. (C) * This answer is correct; it strengthens the researcher�s reasoning. To be sure, apply this principle to the facts from the passage. A small cavity is only potentially harmful and filling a cavity is definitely harmful; therefore, a small cavity that poses no imminent threat should not be filled. If you accept this principle, you must reach the same conclusion as the dental researcher. (D) This is an incorrect answer because it does not support the reasoning of the passage. Temporary relief is irrelevant and inapplicable to the passage. In this example the procedure is not harmful and provides a benefit to the patient. (E) The dental researcher did not qualify his argument with a caveat� that, if the dentist is unable to keep a harmless cavity under constant surveillance, then the dentist should fill it. 5. Number of codfish- This is a weaken the argument question. The author provided us with the following facts. With the increase of seals, there was a decline in cod. Seals don�t eat much cod. Therefore, seals are not causing the decline of the cod population. Any ideas? Maybe they compete for food. (A) This answer has a neutral effect. It does not weaken the author�s argument. It provides new information that is not relevant. (B) This is an obviously incorrect answer; it strengthens the author�s argument. It does not even address the author�s argument. (C) This is an obviously incorrect answer; it also strengthens the author�s argument. (D) * This answer is correct; it weakens the author�s argument. It is evidence that, although harp seals do not eat codfish, harp seals eat the fish that codfish depend upon for food. Therefore, harp seals negatively affect the codfish population. (E) This is an incorrect answer; it does not weaken the author�s argument. It supports the author�s argument by suggesting that there is no temporal correlation

Page 3: 1 1 1 1 1 1 - dl.keywin.orgdl.keywin.org/5/4/54bc444545de55e9a8148ce94764a168.pdf · 1 1 1 1 1 1 Analysis of the ... Why does the LSAT test your ability to evaluate an argument? Because

Get Prepped! PrepTest 28

3

between harp seal increase and codfish decrease. 6. Hospital auditor- This is a point at issue question. You need to identify the conclusion made by the first speaker. The second speaker always disagrees, so you need to figure out on what basis they do so. Do not confuse the conclusion with either a fact or a premise from the passage. The main point at issue is whether the hospital has complied with the wishes of the Rodriguez family regarding the donated funds. (A) This is an incorrect answer; �completely eliminating� is not the point of contention. The test writers worded this answer to look correct, but, sadly, it is wrong. (B) This is an incorrect answer; the main point at issue is not the adequacy of the treatment received by patients. (C) This is an incorrect answer; there is no question that the Rodriguez family clearly stated how the funds were to be spent. (D) * This answer is correct; the entire dialogue revolved around whether the hospital administrator has conformed to the Rodriguez family�s wishes. The auditor stated that the administrator has misspent the funds, while the administrator disagreed. (E) This is close, but no cigar. What the family �anticipated� is not the main point. Maybe they anticipated this, maybe they didn�t. But they clearly stipulated what the funds should accomplish. If you liked and understood the distinction between (D) and (E), you may have a career waiting for you in trusts and estate law. 7. Hospital auditor- This is an �A responds to B� question. Now we don�t care about the point at issue, we care how the second speaker played off the first speaker�s argument. Identify how the administrator responds to the auditor. Pay attention to the structure (pattern of reasoning) of the hospital administrator�s argument. (A) This is an incorrect answer. It is too debatable and overly broad to be correct. The administrator did not concede the �broad conclusion� nor did he point out a �minor qualification.� (B) This is an incorrect answer; the hospital administrator did not appeal to subjective or idealistic principles by arguing that things �ought� to be different. (C) This is an incorrect answer. If anything, the auditor makes this argument. (D) This is an incorrect answer; the administrator never conceded a violation, nor mentioned intent. (E) * This answer is correct; it clearly and accurately reflects the administrator�s method of reasoning. The auditor interpreted the term �patients� to mean current patients of the clinic with previously diagnosed neurological disorders. The administrator interpreted the term �patients� as to include future patients with, as of yet, undiagnosed neurological disorders. The underlying assumption was that research doesn�t minimize suffering. 8. Generally speaking- This is a resolve the paradox question. Identify the fact that most helps to explain the similarity between alfalfa and non-nitrogen fixing plants. First paraphrase the passage into argument form. Plants that deplete nitrogen do not grow well year-after-year in the same field. Alfalfa does not deplete nitrogen, but it too does not grow year-after-year in the same spot. Maybe something other than nitrogen depletion effects alfalfa�s ability to grow well year-after-year in the same field. (A) This is an incorrect answer; it introduces extraneous information of no importance. (B) This is an incorrect answer; it looks promising, but it is only a restatement of fact from the passage. (C) * This answer is correct; it identifies a factor, apart from nitrogen depletion, that explains why alfalfa does not grow well year-after-year in the same field; alfalfa produces a toxic chemical in the soil that affects only alfalfa. (D) This is an incorrect answer; it does not identify a factor other than nitrogen depletion that affects alfalfa�s ability to grow in the same spot. This answer states that nitrogen will not increase in the presence of a certain bacteria. So what? Alfalfa does not deplete nitrogen. (E) This is an obviously incorrect answer. It is too time-consuming to enumerate all the reasons why this answer is wrong. 9. Political commentators- This is a flawed reasoning question. These are similar to weakening questions. Typically, the author has overlooked an important fact, made an unwarranted assumption, or reached an invalid conclusion. It helps to paraphrase the passage into a premises and a conclusion. Once in simplified form the mistake usually becomes obvious. The political commentators say �appeasement.� Most people disagree with the political commentators. Therefore it is not appeasement. The author mistakenly equated the majority opinion with being correct. (A) This is an incorrect answer. The term policy is not ambiguous. (B) The identity of the

Page 4: 1 1 1 1 1 1 - dl.keywin.orgdl.keywin.org/5/4/54bc444545de55e9a8148ce94764a168.pdf · 1 1 1 1 1 1 Analysis of the ... Why does the LSAT test your ability to evaluate an argument? Because

Get Prepped! PrepTest 28

4

commentator�s is irrelevant. Only their view is at issue. (C) * This answer is correct; it shows the author�s mistake. He overlooked the obvious; that it is possible for the majority to be incorrect. (D) This is an incorrect answer; the claim that political commentators are mistaken was a conclusion, not a premise. (E) This is an incorrect answer; it misses the author�s mistake. It incorrectly assumes that if something is true for one person, then it is true for all persons. The author mistakenly assumed that if most of the people think something is true, then that actually is true. 10. Principle of economics- This is a miscellaneous question type. We are told a principle. Now we need to find a new set of facts that correctly apply the principle. (A) This is a wrong answer. The passage stated that a nation can experience growth under these specific circumstances. This answer states that a nation will experience economic growth. (B) This is an incorrect answer; It confuses what is possible with what is certain. The passage stated that slight skepticism makes economic growth possible. This answer concludes that slight skepticism makes economic growth certain. (C) This contradicts the passage. The passage stated that economic growth required a balance of confidence and skepticism. Therefore, growth is impossible when there is only confidence or only skepticism. (D) * This answer is correct; it is a valid application, directly supported by the conditions set out in the passage. See the explanation to answer choice (C). (E) This answer is incorrect for the same reason as choice (B). The right balance of skepticism and confidence does not guarantee economic growth; it simply makes it possible. 11. Sharks have- This is a make a conclusion question. Based on the passage, four of the five answers are valid conclusions. Select the one that cannot be true or that is ambiguous. (A) * This answer is correct; it cannot be true based on the passage. The author explicitly stated that sharks have �a greater resistance to cancer than any other organism.� Thus, it is impossible for any other organism to resist cancer as well as sharks. (B) This is an incorrect answer; it could be true based on the passage. The author stated two facts: Sharks have the highest percentage of cartilage, and sharks have the lowest rate of cancer. The only thing for certain is that the organism with the highest rate of cancer has less cartilage than sharks. (C) This answer is incorrect; it could be true. The passage does not preclude the possibility that cancer-fighting substance is present in other organisms. (D) This answer could be true. The fact that other effective therapies exist does not contradict the fact that shark cartilage is the best therapy. (E) This answer is wrong because it could be true. The author addressed only sharks� immunity to cancer. He or she did not state that sharks have the most efficient immune system. 12. People who say- This is an identify an assumption question. Remember, you will not find the answer in the passage; so, read between the lines for what isn�t there in black and white. The author argued that farmers with flat land do not build terraces to prevent erosion by water; farmers in Dooney County build terraces; therefore, the land in Dooney county is not flat. To reach this conclusion, the author had to assume something about the reason why farmers in Dooney County build terraces. (A) This is an incorrect answer; to reach this conclusion, the author did not have to assume that the only cause of soil erosion is water. (B) * This is the correct answer. To reach the conclusion that Dooney county is not flat, the author had to assume that the terraces in Dooney county were constructed to prevent soil erosion because on flat land soil erosion is not a problem. (C) This is an incorrect answer; it is not necessary to assume that the terraces actually prevent erosion. It is only necessary to assume that the farmer intended the terraces to prevent erosion. (D) This is an incorrect answer; it partially restates a premise from the passage: �On flat land, soil erosion by water is not a problem.� (E) This is an incorrect answer; the author�s conclusion depends on there being at least one terrace on a farm. It does not depend on the assumption that terraces exist exclusively on farms. 13. People who say- This is an identify parallel reasoning question. This is the only time you will see a parallel reasoning question as part of a two-question passage. First, paraphrase and/or diagram the argument from the passage. Second, disfavor an answer choice that contains the same wording as the passage. The author argued that if land is flat, then soil erosion is not a problem. Farmers whose land is

Page 5: 1 1 1 1 1 1 - dl.keywin.orgdl.keywin.org/5/4/54bc444545de55e9a8148ce94764a168.pdf · 1 1 1 1 1 1 Analysis of the ... Why does the LSAT test your ability to evaluate an argument? Because

Get Prepped! PrepTest 28

5

flat do not build terraces to prevent erosion. Farmers in Dooney County build terraces. Therefore, Dooney County is not flat. A simple diagram to represent the pattern of reasoning may look like this: If F Then no SE. If F no T�s to prevent SE. In DC are T�s. Thus, DC not F. (A) This is an incorrect answer, it does not follow the reasoning of the passage. A simple diagram of this argument may look like this: If P(white) Then S and P again. Thus, P(dark blue). Compare this diagram to the diagram of the argument from the passage and you see that they are dissimilar. (B) This is an obviously incorrect answer; dismiss it outright. (C) * This answer is correct; this is a similar pattern of reasoning. A simple diagram may look like this: If a lot of E then no TFA. If a lot of E then no use RX to help FA. J use RX. Thus J no a lot of E. Compare this diagram to the diagram of the argument from the passage and you see that they clearly are similar. (D) This is an obviously incorrect answer; dismiss it outright. (E) This is an obviously incorrect answer; dismiss it outright since there is a disconnect in the cause and effect. 14. The axis of- This is make a conclusion question. Based on the given facts or conditions, reach a logical and carefully tailored conclusion. (A) Close, but this is incorrect; it is too broad a conclusion. If Mars had a large moon nearby, then it would have a stable, moderate tilt. However, there is no basis to conclude that a moderate tilt, in and of itself, is sufficient to support life on mars. (B) * This answer is correct; this conclusion is directly and completely supported by the facts of the passage. Without a stable axis a planet cannot support life. If the moon left earth�s orbit, then earth would be without a stable axis. Therefore, earth would no longer be able to support life. (C) This is an incorrect answer; there is no basis to conclude that a moderate tilt, in and of itself, is sufficient to support life on any planet. (D) This is an incorrect answer; it is too broad of a conclusion. While the passage makes clear that a sizeable moon has an effect on the tilt of a planet. It does not discount the possibility that other gravitational influences do exist (i.e. the Sun). (E) This is an incorrect answer; it is too broad a conclusion. The reason that Mars cannot support life is because it has an unstable tilt. The number of moons is not determinative of whether a planet can support life. The amount gravitational force is the determinative factor. 15. Town of Springhill- This is a strengthen the argument question. The author concluded that Springhill�s monthly flat fee, charged for any amount of water below a certain usage threshold, discourages conservation. The correct answer should address the threshold level. (A) This is an incorrect answer; it provides additional information that is irrelevant to the author�s argument. Whether the Springhill authorities enforce water emergency laws is not a factor that affects operation of the payment scheme. (B) This is an incorrect answer. If the threshold were raised, the problem would become worse. (C) * This strengthens the author�s argument. If the threshold is much higher than necessary, then households can waste water without hitting the thresholds. Thus, the households have no incentive or disincentive to use only the amount of water they need. (D) This is an incorrect answer; the threshold for water use in other towns is irrelevant, or weakens the conclusion. (E) This is an incorrect answer; the amendment process is irrelevant to the issue of whether or not the threshold is too high. 16. Poppy petals- This is a make a conclusion question. The first step is to identify the facts. When pollinated, release substance and wilt in 2 days. If not pollinated, substance not release, wilt in 1 week. Cutting unpollinated flower releases substance. Therefore, we can conclude that a cut poppy wilts in 2 days. (A) This is too broad to be a conclusion. The passage makes clear that insects are attracted to poppy petals. It does not specify that the insects are exclusively attracted to full, healthy petals. (B) * This is the correct answer; the passage most strongly supports this conclusion. The substance released during pollination causes the poppy to wilt. To cut the poppy releases the same substance. Therefore, to cut the poppy causes the poppy to wilt. (C) This is an incorrect answer. �All plants� is outside the scope. (D) This is new information. While this answer could be true, it is not a conclusion derived from these facts. (E) This answer is a logical assumption, but it is not a valid conclusion directly supported by facts from the passage. 17. When a community- This is a resolve the paradox question. Remember, overall economic growth depends on new money coming in from outside the community, not simply redistributing the money that is

Page 6: 1 1 1 1 1 1 - dl.keywin.orgdl.keywin.org/5/4/54bc444545de55e9a8148ce94764a168.pdf · 1 1 1 1 1 1 Analysis of the ... Why does the LSAT test your ability to evaluate an argument? Because

Get Prepped! PrepTest 28

6

already there. (A) This is an incorrect answer; it does not explain the discrepancy. The important issue is whether or not shoppers at the mall are spending new money, from outside the community. Only new money contributes to overall growth. (B) This is an incorrect answer; it does not explain the discrepancy. If this scenario were true, the increase in economic growth would closely mirror the total amount of economic activity at the mall because every dollar spent at the mall would come from outside the local economy. (C) Discussing jobs, this answer is wrong. It fails to address the money issue. (D) * This is the correct answer, it explains the discrepancy. The total amount of economic activity at the mall is greater than the overall economic growth (influx of new money). Therefore, some of the money spent at the mall is not new money; it came from inside the old local economy at the expense of other local merchants. This is the so-called �Wal-Mart effect.� (E) This answer is wrong. Job creating is irrelevant to explaining the discrepancy, at least as it is conceived in this passage. 18. Essayist- This is a make a conclusion question. Essential to making a conclusion is knowing the facts. We are told that science can be changed. We must decide if change is warranted, taking into account price. A list of examples are provided to waste your time. Massive interventions are costly and change character. Therefore: ? (A) Possible. (B) Maybe. (C) This is an incorrect answer; it is a fact, not a conclusion. This answer simply restates the first line of the passage. (D) This is an incorrect answer; the imposition of restrictions, if they were not massive interventions, would not necessarily be very costly. (E) * Maybe. (A) This is an incorrect. The author stated that massive interventions would change the character of science. The author remained silent about whether the changes should or should not occur. (B) The author did not conclude that closer regulation, without massive interventions, would change the character of science. (E) * This is our last chance. The author would agree that change should be made, if it is warranted. Before making changes we are told to consider the impact of the cost and the change of the character of science. 19. The postmodern view- This is a flawed reasoning question. Identify the reason why the author�s logic is doubtful. (A) * Maybe. (B) Maybe. (C) There is no emotional appeal. (D) There is no ambiguity in this term. (E) This is incorrect; the author�s failure to provide examples did not automatically render his argument invalid. (A) * The author based an objective conclusion on a subjective premise. In other words, he mistook opinion for fact. Examine the last two lines: �Also, the belief in order has given way to a belief in the importance of irregularity and chaos. (Conclusion) It follows that we inhabit a world full of irregular events�� (B) This is very attractive. The terms �universality of truth� and �universal truth� are used. But this is not an ambiguous use of the term �universal.� The term is used in two clearly different senses. 20. If the economy- This is a make a conclusion question with a twist. Choose the one answer that cannot be true, based on the passage. In other words, the correct answer must be the opposite of a valid conclusion. To make this argument clearer, edit it by replacing �although� with �and,� and delete both �But� and �Fortunately.� Now it is easier to see that a diagram of this passage would look like the following diagram. If W ! $C AND UE+. (But only get UE+ if Investment decreases.) Since Investment is not decreasing, (then UE is not rising) so the economy is not W. (A) * This is the correct answer; it must be false because it contradicts the passage. The author explicitly stated that investment is not decreasing. Since investment is not decreasing, then the economy is not weak. (B) This is an incorrect answer. Based on the passage, it could be true that unemployment rises and prices remain constant. (C) This is an incorrect answer; it could be true. If investment decreases, then unemployment rises. If unemployment rises, then it is possible that the economy is weak. So, it is possible that if investment decreases then the economy is weak. (D) This is an incorrect answer; it could be true that prices are remaining constant. Based on the facts, we don�t know what prices are doing. (E) This is an incorrect answer; it could be true that the economy is not weak. 21. Psychologist- This is an identify an assumption question. You know the drill by now. The facts are: Astrologers claim horoscopes determine personality. We found two people born at exact same time. (Insert assumption here.) Therefore, horoscopes do not determine personality. What must the psychologist

Page 7: 1 1 1 1 1 1 - dl.keywin.orgdl.keywin.org/5/4/54bc444545de55e9a8148ce94764a168.pdf · 1 1 1 1 1 1 Analysis of the ... Why does the LSAT test your ability to evaluate an argument? Because

Get Prepped! PrepTest 28

7

believe in order to reach this conclusion? (A) This answer looks promising, but it is an attractive wrong answer. The answer is attractive for two reasons; it supports the psychologists� position, and it is likely to be true. However, the author did not have to make this assumption in order to reach his or her stated conclusion. Tricky stuff. (B) This is another good-looking answer, and it too is wrong. The author did not hinge his or her conclusion on this assumption. (C) * This answer is correct; the author assumed that location of birth was irrelevant to a person�s astrological sign. Therefore, the author concluded that the person in Toronto had the same exact horoscope as the person in New York. If the two persons had different horoscopes, then astrologists could still say that the different horoscopes explain the differences in their personalities. (D) This is a wrong answer; if true, it might support the author�s argument, but it is not essential to the author�s conclusion. (E) This is an incorrect answer. Obviously this fact is true, but it is not a building block of the author�s argument. 22. Under the influence- This is a make a conclusion question. (A) This is an incorrect answer. The passage mentioned the inclusion of humorous material; however, it explicitly stated that the reason for a lack of substance is an emphasis on illustration and graphic design, not the humor. (B) * This answer is correct; the passage stated that lack of substance causes short-lived books. Therefore, you can safely conclude that substance is important to a book�s longevity. (C) This is an obviously incorrect answer; though it may be factually true. (D) This answer is incorrect; the passage does not provide enough information to support this conclusion. (E) This answer is incorrect; the passage never touched on the issue of popularity. 23. Further evidence- This is a strengthen or weaken the argument question with a twist. The correct answer is the lone choice that does not weaken the argument, but instead strengthens it or is irrelevant. This one is pretty subtle. The facts are: Depressed people have less front lobe activity. Happy people have more front lobe activity. Therefore, ones disposition is cause by the front lobe activity. Obviously there is a cause and effect flaw. So keep your eyes open for that. (A) * This answer is correct; it strengthens, not weakens, the argument. The author concluded that psychological problems are caused by decreased activity in the left lobe of the brain. If the way they fix this psychological problem is to increase the activity in the left side of the brain, then decreased activity must be the cause of depression. The medicine causes the brain to be more active and this eliminates the depression. (B) This is an incorrect answer. The author argued that decreased brain activity is the cause and that depression the effect. This answer states that depression (which leads to excessive sleep) is the cause and decreased brain activity is the effect. (C) This weakens the conclusion by denying that there is a correlation between brain activity and disposition. (D) This answer is incorrect; it states that activity in the limbic system (which is not the front lobe), causes lobe activity AND emotions. It is the cause and the other two are the effect. (E) This is an incorrect answer; it suggests that a decrease in brain activity is only an effect of depression, rather than the cause. The cause (interaction) creates the effect (more lobe activity.) 24. We ought to- This is a strengthen the argument question. Maybe they are getting harder, or maybe you are getting more tired, but just hand on for a few more. The correct answer is the choice that, if added to the premises from the passage, logically leads to the stated conclusion. (A) * This is the correct answer; if you place this answer into the author�s argument it logically supports his or her conclusion. We should only pay attention to a work of art�s intrinsic properties. What artwork symbolizes involves only extrinsic properties. Therefore, we should not pay attention to what artwork symbolizes. (B) This answer, while true, is incorrect. The author�s argument did not hinge on this premise. It turned on the difference between extrinsic and intrinsic properties. (C) This is an incorrect answer because it merely restates the first line of the passage. (D) This might be a conclusion based on the author�s argument, but it certainly is not an assumption. (E) �[R]elates the work to itself� is obviously incorrect. 25. McKinley- This is a miscellaneous type question. The first speaker said that the new drug will have some effect (maybe it will cause green dots) that will make the researcher aware of who is getting the drug vs. the placebo. The second speaker starts yammering about the �outcome of the study will be�� But the

Page 8: 1 1 1 1 1 1 - dl.keywin.orgdl.keywin.org/5/4/54bc444545de55e9a8148ce94764a168.pdf · 1 1 1 1 1 1 Analysis of the ... Why does the LSAT test your ability to evaluate an argument? Because

Get Prepped! PrepTest 28

8

outcome was irrelevant to the limited issued discussed by the first speaker. Why did the second speaker in the passage misunderstand the first speaker�s argument? (A) Close, but wrong. The first speaker presumed that a double blind study was the �most effective� way, not the only effective way, to test new drugs. (B) This is an incorrect answer. Engle interprets McKinley as saying what the outcome will be in advance, but not specifically whether or not the drug will be effective. (C) This is wrong. The first speaker assumed that the placebo will have no noticeable side effects on the patients� bodies. (D) * This is the correct answer; it describes why Engle misunderstood McKinely. He though McKinley was talking about the therapeutic effects, but he more likely meant the side effects (green dots). (E) This is incorrect; there is no evidence that shows Engle misunderstood McKinely for this reason. 26. Modern navigation- This is a strengthen or weaken the argument question. Identify the answer choice that does not strengthen the argument, i.e. it either weakens it or is neutral. (A) This answer is incorrect; if true, it would strengthen the author�s claim that passengers� electronic devices are risky. (B) This answer is incorrect for the same reason as the previous choice (A). (C) This is an incorrect answer; it tends to prove that portable electronics are the specific cause of interference with navigation systems. (D) This is an incorrect answer; it establishes that at least some portable devices are close enough to cause interference. (E) * This answer is correct; when portable devices first appeared is irrelevant to whether or not they interfere with navigation systems.

Page 9: 1 1 1 1 1 1 - dl.keywin.orgdl.keywin.org/5/4/54bc444545de55e9a8148ce94764a168.pdf · 1 1 1 1 1 1 Analysis of the ... Why does the LSAT test your ability to evaluate an argument? Because

Get Prepped! PrepTest 28

9

(322-325)

2 2 2 2 2 2 A review of Analytical Reasoning

Important strategies- The most important single strategy in the Puzzle Games is to use the correct diagram. This is the best way to keep the members correctly organized, and usually it is the only way. More than 95% of the LSAT puzzles in the last 10 years could be answered using a diagram, and most puzzles were virtually impossible to answer without using a diagram. The second vital strategy in the Puzzle Games is to get adept at discovering the extra conclusions that are the key to unlocking the puzzle. Other strategies include using the correct series of steps to make sure you efficiently and correctly summarize each condition, and using good bookkeeping so as not to overlook a crucial condition. If you have time, read every answer! If your diagram is correct, there can never be two possible answers! Finally, there is the condition violator answer elimination technique, which always is a good short cut to avoid diagramming. More on puzzle types. There are several main puzzle types. They are the line, the matrix, the multiple-line, and the subset. There are several minor derivatives puzzle types: heavier than lines, the networks, and the organization charts. Finally, there are hybrid possibilities, which will combine elements of two or more puzzle types. Don�t let the possible variations phase you. If you can make a class schedule from the hundreds of possible courses offered by your school, you have the ability to master any of these puzzles. Just like you should do in reading comprehension, you should skim all four games to see which has the most questions and also to determine what kind of puzzles will be used for each one. Start with the easier puzzles, especially if you have trouble completing the Puzzle Games sections. Question Set Difficulty:

The first set was really easy to set up and answer. The second puzzle set was extraordinarily difficult. It is rare to see a puzzle this complex. The third puzzle was fairly simple to set up, but probably of average difficulty. The fourth puzzle was pretty easy to set up, and pretty easy to answer. In retrospect, we see that the test-makers had to compensate for two very easy puzzles by adding a very difficult puzzle. Because the second puzzle was so complex, most takers probably drained too much time away from doing the two later easier puzzles. This is a very good lesson to learn. Never spend more than your time budget on a question or puzzle set.

Page 10: 1 1 1 1 1 1 - dl.keywin.orgdl.keywin.org/5/4/54bc444545de55e9a8148ce94764a168.pdf · 1 1 1 1 1 1 Analysis of the ... Why does the LSAT test your ability to evaluate an argument? Because

Get Prepped! PrepTest 28

10

Overview of puzzle set #1- Racehorse line-up This was the simplest of simple lines. There is no way anyone should have diagrammed this

incorrectly. Here is the diagram you should have after working through the conditions. __ __ P __ __ __ 1 6 Condition 1: L _ K or K _ L Condition 2: M < N Condition 3: NK Condition 4: P = 3 You can see that N will never be in position 1 and M will never be in position 6 Also, neither K nor L can ever be in positions 1 or 5, since doing so would conflict with P in 3. This condition affecting K and L will be very important for solving this puzzle. 1. This was a simple answer elimination question. (A) violated condition 1, (C) violated 4, (D) violated 3, and (E) violated 2. (B) * is the correct answer. Re-draw the graph if it helps you see the relationships. 2. To start, don�t bother checking for the truth of �2.� Since all five choices contain it, it is obviously true. We can eliminate (B) since P must be in 3. We can eliminate (A) because K can�t be in 1, since L is not allowed to go into 3. Since �4� is in the three remaining choices, ignore it. (D) Can K be in 5? No. (E) * Can K be in 6? Yes. 3. With a CANNOT be true question like this, it may be easiest to quickly work each answer choice, but try to eliminate some easy ones. (A) In the last question we were told K can be in 2, so we don�t need to do this work again. (B) Same analysis. Since K and L are mirror images, if K can go in 2, then L should be able to as well. (C) Now place M in position 1. Then place K in 2 and L 4. Then N and O can go in 5 and 6. (D) Try placing M in 5. This would require N to be in 6. Then K and L could go in either 2 or 4, and O can go in 1. (E) * Place O in 2, this means that K and L must go to 4 and 6. Now M must be before N, so M must go into 1, leaving only 4 for N. But if N is put into 4, it is going to be next to K, which is prohibited. This cannot be true. 4. We have done enough work now that the insights gained from earlier questions will help us. (A) In the previous question, we found that K and L can go into 2 and 4, or into 4 and 6. (B) * Same analysis. The common element is that K or L goes into 4, no matter what. (C) Since we saw that K or L may be in 2, this would indicate that M and N are not required to be there. (D) Actually, O can go into 5. (E) K or L can be placed in 6 too. (Figure 1) M K/L P L/K N/O O/N or M N P K/L O K/L Fig. 1 5. How might you solve this? Use any insights you have already developed from the prior questions. (A) We can see the L can be next to N, see figure 1. (B) Also, M and K can be next to each other. (C) * In figure 1 M is always in position 1, while O is in 5 or 6. (D) & (E) See figure 1.

Page 11: 1 1 1 1 1 1 - dl.keywin.orgdl.keywin.org/5/4/54bc444545de55e9a8148ce94764a168.pdf · 1 1 1 1 1 1 Analysis of the ... Why does the LSAT test your ability to evaluate an argument? Because

Get Prepped! PrepTest 28

11

Overview of puzzle set #2- Languages This was an extremely difficult puzzle. The biggest problem was there were very few anchors, so

you had to solve each problem individually, there were no short cuts. Hopefully you did figure out to use a matrix and got at least this far. Condition 1: R=1 Condition 2: S=2 Condition 3: T=2 Condition 4: Y=3 Condition 5: If L or P, no G Condition 6: If G, then H G (if G then H) XXXX XXXX H Yes L (If L, not G) Yes P (if P, not G) Yes (learn 1-3 each) R =1 S = 2 T = 2 Y = 3

Maybe you saw that since R can only have 1, that G could never fill the R slot, because H would not be allowed to fill R as well. This was the first warranted conclusion. Next, turn your attention to Y. Exactly three must learn Y. So try an experiment. What if G spoke Y? Then H would also have to speak Y. Now 1 more person would need to speak Y, and only L and P remain. But neither L nor P can speak the same language as G, so we can conclude two things. First, that it is impossible for G to learn Y. Second, this means that H, L, and P must all learn Y. This was a vital warranted conclusion. Answering the questions would be virtually impossible otherwise. The questions still require a lot of hard work running through the conditions. G will always learn either S or T, and so L and P will not learn whatever G learns. 6. This one was hard. Maybe you chose to �what if� each of the proposed answers. Maybe you noticed that three answer choices dealt with R, S, and T, and so that would alert you to consider how that pattern would fit. But probably you didn�t notice the patterns. So, just work to solve it. (A) Our graph shows that L must learn Y, and so can�t also learn three more languages, R, S and T. If L did learn R S and T, then this would force G to do Y, but three folks must speak Y, and G may only be paired with H. This same reasoning eliminates (C) and (E). (D) * If H does learn S T and Y, this allows G to either do S or T. Then L and P can learn R and anything that is left over. See below, it is a bit confusing; these are the permutations. (Figure 1) G (if G then H) XXXX Yes | or no Yes | or no XXXX H No Yes Yes Yes L (If L, not G) ? No | or yes No | or yes Yes P (if P, not G) ? No | or yes No | or yes Yes (1-3 each) R =1 S = 2 T = 2 Y = 3 Fig. 1 7. Again, a very hard question but not as hard as the last one. We know to ignore G, since he can�t learn R. If L, for example (but it also could be P), learns three, how is he going to avoid running into G? L will definitely need to learn Y, since Y requires three speakers. Since G can�t do R, then L can do R. Now L (or P) can either do S or T. So the two possibilities are (R S Y) or (R T Y). Either way, your graph looks like this. (Figure 2) So L must speak either S or T. (B) * is correct.

Page 12: 1 1 1 1 1 1 - dl.keywin.orgdl.keywin.org/5/4/54bc444545de55e9a8148ce94764a168.pdf · 1 1 1 1 1 1 Analysis of the ... Why does the LSAT test your ability to evaluate an argument? Because

Get Prepped! PrepTest 28

12

G (if G then H) XXXX ? ? XXXX H no ? ? Yes L (If L, not G) Yes maybe maybe Yes P (if P, not G) no ? ? Yes (1-3 each) R =1 S = 2 T = 2 Y = 3 Fig. 2 8. This is another hard one because four are true and one is not. Perhaps you realized you should look for an answer choice containing Y, since whoever speaks R also happens to speak Y. Remember, G can�t speak R or Y, which forces the other three to speak Y. So answer choice (C) * is correct, since it says that they do not learn Y. 9. This question is really hard to understand. Make this question more understandable by plugging in an answer choice. Remember, P and L can share the same analysis. Determine if P can speak S and Y without anyone else. We know P must speak Y, and if P speaks S, then G does not speak S, and so must speak T, which means H thus speaks T, filling the two spots for T. (Figure 3) G (if G then H) XXXX No Yes XXXX H ? ? Yes Yes L (If L, not G) ? ? No Yes P (if P, not G) ? Yes No Yes (1-3 each) R =1 S = 2 T = 2 Y = 3 Fig. 3

See the two question marks in the S column? One of them must speak S in order to reach the two speaker requirement, so P cannot be the only one to speak S. So (B) * is the correct choice, P can�t speak S alone. Why not (A)? This is kind of a trick question. This question doesn�t say that G can�t speak S, rather it asks who can speak both S and Y. If G were to speak S, and H were to speak S, then (A) would be in accurate list of those who could speak Y AND S. Tricky? Yes, it was very tricky. 10. These questions tend to be a bit easier. Let�s plug it in and see. (Figure 4) There are only two languages G can speak, S and T. This means H speaks them too. Also, H has now reached the maximum three languages, so note this in the R column. The only thing we don�t know is if L or P is in the R column. Looking at our graph, we see that all the answer choices are false, except (A) *, P might learn R. G (if G then H) XXXX Yes Yes XXXX H No Yes Yes Yes L (If L, not G) ? No No Yes P (if P, not G) ? No No Yes (1-3 each) R =1 S = 2 T = 2 Y = 3 Fig. 4 11. This one required a more general overview. What must be true, based on our initial diagram? First, G must not speak R or Y. Second, H, L, and P must speak Y. (A) Based on what we know, H may speak more, fewer, or the same number of languages as P. (B) * If we look at our graph, we realize that if G speaks a language, H must speak it also. Add to this the fact that H also has a one-language advantage over G, because H must speak Y, and you see that H must always speak one more language than G. (C) G could speak two, and you can find a permutation that allows L to speak only one. (D) P and L are totally interchangeable, so you should have known one may, or may not, have more languages than the other. (E) P could speak three, while H could speak two.

Page 13: 1 1 1 1 1 1 - dl.keywin.orgdl.keywin.org/5/4/54bc444545de55e9a8148ce94764a168.pdf · 1 1 1 1 1 1 Analysis of the ... Why does the LSAT test your ability to evaluate an argument? Because

Get Prepped! PrepTest 28

13

12. If H learns two, and we already know that one is Y, what can we deduct? We know that H will not learn R, because then G would have no H to accompany in S or T. So H, and by implication, G, must learn one language, S or T. Let�s see if that is enough to find an answer. (A) and (C), remember L and P should be interchangeable, so both these choices fail. (B) G may learn S or T. (D) * Yes, we know that H cannot learn R, but rather must learn S or T so as to allow G to learn a language. (E) P can be placed in S, if H and G are in T.

Page 14: 1 1 1 1 1 1 - dl.keywin.orgdl.keywin.org/5/4/54bc444545de55e9a8148ce94764a168.pdf · 1 1 1 1 1 1 Analysis of the ... Why does the LSAT test your ability to evaluate an argument? Because

Get Prepped! PrepTest 28

14

Overview of puzzle set #3- Inspection schedule This was a much easier schedule matrix. When you see days and AM/PM in the set-up, you know

it is a matrix. Condition 1: Hotels not on W Condition 2: G < J Condition 3: GS Condition 4: IF Z AM, then L is AM Note that only S, V, or Z can fill Wends. Here is the basic schedule. (Figure 1) The other three conditions we can�t put on our graph yet. M T W (no G J or L) AM PM Fig. 1 These are all the possible permutations, if you chose to graph them all: M T W (no G J or L) AM (G or L + 1 restaurant) (G J or L +1 restaurant) S or V or Z PM (G J or L +1 restaurant) (J or L +1 restaurant) S or V or Z 13. As is normal, use condition violator answer elimination for the first question. (A) violates 3. (B) violates 1. (C) violates 2. (E) violates the only condition left, 4. (D) * wins by default. 14. See if any obvious ones can be eliminated. (A) is gone, it violates 4. (B) violates 2, and our diagram above. (D) and (E) both have the same problem. Diagram it out and you see that these would put G and S on the same day, violating 3. (C) * wins. 15. You can try to solve this different ways. The easiest way is to try to put these pairs on a different day to see if they can work. A quick example using (A); See, G and J don�t need to be on Monday, because we just have shown that they may go on Tuesday. (Figure 2) (B)* This is correct. (C) G must be before J. (D) See figure 3. (E) See figure 4.

L G Z S J V

Fig. 2

G S V J L Z

Fig. 3

G V S J L Z

Fig. 4 16. Try to diagram this. Note that G is related to two other members. First, G and S can�t be on the same day. Second, G must come earlier than J. Focus on G and J. Since G is to be on T, yet must come before J

Page 15: 1 1 1 1 1 1 - dl.keywin.orgdl.keywin.org/5/4/54bc444545de55e9a8148ce94764a168.pdf · 1 1 1 1 1 1 Analysis of the ... Why does the LSAT test your ability to evaluate an argument? Because

Get Prepped! PrepTest 28

15

(who may not be inspected later than T PM), then G must be T AM and J must be T PM. So eliminate any answer choices that don�t show J on T PM, (C) and (D). (Figure 5) M T W (no G J or L) AM (L or 1 restaurant) G S or V or Z PM (L or 1 restaurant) J S or V or Z Fig. 5

L must be on M. This allows us to eliminate (E), since having L on W would violate condition 4

and our current diagram. Now consider how L is related to other members. If Z is AM, than L is AM. This means that Z may not be on M AM. Also, if Z is on W AM then L must be on M AM. Now it gets trickier. Since Z cannot be on M AM, either L, S or V may be there. Looking at our two remaining answer choices, if (A) L is M PM, and V is W PM, then either S or Z is W AM, but Z can�t be W AM if L is not in the M AM, so this means S must be W AM, leaving Z to fill M AM, but we know this isn�t allowed by condition 4. So (B) * is the only choice that will work. 17. If S is M AM, then G must be on T, and working with some of the analysis from the previous problem, we know that if G is on T, it must be T AM, and J must be T PM. This means that L is left to fill M PM. Since L is in M PM, Z is not permitted to be in W AM, and so V is W AM and Z is W PM. (D) * is correct. 18. If Z is W AM, we know that L must be T AM. If G is M AM, J can be M PM or T PM. S may not be on M PM, but V may. A diagram of the possibilities is shown in figure 6. M T W (no G J or L) AM G L Z PM J or V J or S or V S or V Fig. 6 Looking at our choices, (A) & (B) J may be before or after L. (C) No, J is inspected before S. (D) * Same analysis. (E) L may be before or after V.

Page 16: 1 1 1 1 1 1 - dl.keywin.orgdl.keywin.org/5/4/54bc444545de55e9a8148ce94764a168.pdf · 1 1 1 1 1 1 Analysis of the ... Why does the LSAT test your ability to evaluate an argument? Because

Get Prepped! PrepTest 28

16

Overview of puzzle set #4- Voting Although this looked frightening, it ended up being pretty easy. A good diagram will make this

one a breeze. Condition 1: Each votes for 1+, against 1+ Condition 2: 2 against recreation Condition 3: 1 for school Condition 4: 1 for tax Condition 5: F for recreation, against school Condition 6: G against recreation Condition 7: H against tax Graph the initial conditions. TB =1Yes (2 no) SC =1Yes (2 no) RB =2Yes (1 no) F (1Y, 1N, 1?) G(1Y, 1N, 1?) H(1Y, 1N, 1?) Now add in the anchor conditions. TB =1Yes (2 no) No SB =1Yes (2 no) No RB =2Yes (1 no) Yes No F (1Y, 1N, 1?) G(1Y, 1N, 1?) H(1Y, 1N, 1?) Now work with this information and make a warranted conclusion. TB =1Yes (2 no) No SB =1Yes (2 no) No RB =2Yes (1 no) Yes No Yes F (1Y, 1N, 1?) G(1Y, 1N, 1?) H(1Y, 1N, 1?) We only have four boxes that aren�t yet filled, and it is easy to keep track of them. 19. Which could be true? (A) doesn�t work, because one has to be a Yes, and one has to be a No. (B) Just look at the graph. (C) See the analysis for (A). (D) * Maybe. (E) If they both voted Yes on the two bills, then they�d both have to vote Yes for TB, and that just won�t work folks. 20. Let�s use our graph for this one. TB =1Yes (2 no) No Yes No SB =1Yes (2 no) No Yes No RB =2Yes (1 no) Yes No Yes F (1Y, 1N, 1?) G(1Y, 1N, 1?) H(1Y, 1N, 1?) Simple, we completely solved it, now we just look at the diagram. (E) * wins. 21. Another one to plug into the diagram, adding the new anchor. TB =1Yes (2 no) No Yes No SB =1Yes (2 no) No ? ? RB =2Yes (1 no) Yes No Yes F (1Y, 1N, 1?) G(1Y, 1N, 1?) H(1Y, 1N, 1?)

Page 17: 1 1 1 1 1 1 - dl.keywin.orgdl.keywin.org/5/4/54bc444545de55e9a8148ce94764a168.pdf · 1 1 1 1 1 1 Analysis of the ... Why does the LSAT test your ability to evaluate an argument? Because

Get Prepped! PrepTest 28

17

The only uncertainty is which one, G or H, votes for the SB. (A) * F definitely votes for exactly one bill. G may vote for exactly one bill, or maybe two. (B) No, either G or H, one of them, must vote for two bills. (C) This violates our diagram. (D) This violates our diagram. (E) Same analysis. 22. Yet another one to plug in. Since G votes No on RB, he must now vote Yes on the other two. TB =1Yes (2 no) No Yes No SB =1Yes (2 no) No Yes No RB =2Yes (1 no) Yes No Yes F (1Y, 1N, 1?) G(2Y, 1N) H(1Y, 1N, 1?) Our diagram is complete. We see that (C) * is correct. 23. This one is slightly more challenging. What the new condition is trying to say is that two members are going to vote No on the same bills, which means they must also vote Yes the same way. We know that F and G can�t vote the same way, and G and H can�t vote the same way. So G and H must vote the same way. TB =1Yes (2 no) No Yes No SB =1Yes (2 no) No Yes No RB =2Yes (1 no) Yes No Yes F (1Y, 1N, 1?) G(1Y, 1N, 1?) H(1Y, 1N, 1?) Consulting our now complete diagram, (E) * is correct.

Page 18: 1 1 1 1 1 1 - dl.keywin.orgdl.keywin.org/5/4/54bc444545de55e9a8148ce94764a168.pdf · 1 1 1 1 1 1 Analysis of the ... Why does the LSAT test your ability to evaluate an argument? Because

Get Prepped! PrepTest 28

18

(326-333)

3 3 3 3 3 3

Analysis of the Questions: 1. Flavonoids are- This is a make conclusion type question; they are very common. You must make a conclusion based on a given set of facts or conditions. Here, unlike identify a conclusion questions in the reading comprehension section, the answer is not stated in the passage. When answering these questions, keep the following in mind. Limit your conclusion to the given facts and avoid making assumptions. Do not make a conclusion that is logically possible, but not directly supported by the facts in the passage. Do not select an answer that simply restates a fact or condition (remember, the conclusion is not in the passage). If you find two feasible answer choices, select the one that is narrowest in scope. (A) This answer is incorrect; it is too broad a conclusion to be based on the passage. The passage addressed only apples and antioxidants; it did not address all fruits and vegetables, or other factors in the prevention of heart disease. You must base the conclusion on the facts from the passage. (B) This answer is incorrect; you cannot make this conclusion based on the passage. The passage stated that antioxidants, not flavonoids, are known to be a factor in the prevention of heart disease. (C) Initially eye-catching, this contains the correct elements, apples and heart disease. But it is too exact. We don�t know how many apples are required. (D) * This answer is correct; you can make this conclusion based on the passage. Antioxidants prevent heart disease. One specific variety of flavonoid in apples is an antioxidant. Therefore, at least one type of flavonoid prevents heart disease. (E) This answer is incorrect; you cannot make this conclusion based on the passage. The passage did not specify the common causes of heart disease. 2. A number of- This is a vulnerable to criticism question. These questions are similar to those which ask you to weaken the argument, except that you are not adding a new fact, you are merely pointing out an inherent flaw. Typically, the author of the passage has overlooked an important fact, made an unwarranted assumption, or reached an invalid conclusion. You must identify the author�s mistake. Sometimes, it helps to paraphrase the passage into an argument with premises and a conclusion. Once in the form of an argument, the mistake from the passage becomes more obvious. Other times, the author�s mistake will be quite obvious. Here it should be obvious. Paraphrasing the passage: A number of Grandville�s wealthiest citizens have been criminals. Therefore, no wealthy person (at all) should be on the committee. The author reached a conclusion based on a faulty assumption; he concluded something about every member of a group based on the actions of what might be only a few members of that group. (A) There is no cause and effect mistake. (B) The author did not mention, nor consider, a causal or temporal relationship. (C) The author did not mention any of these issues. (D) The author did not use subjective standards; someone is either a criminal or someone is not a criminal. (E) * This is the correct answer; the author generalized that all rich people are dishonest, based on the fact that some rich people are dishonest. 3. Birds startled- This is a resolve the paradox question. Choose the piece of information that explains why the birds fly toward windows instead of foliage. (A) This answer is a bit off the subject; it addresses the behavior of predators, not birds. (B) This answer does not explain the anomaly. This fact is irrelevant, useless, information. (C) This answer is incorrect. Large or small, red or blue, fat or thin; none of these are relevant. (D) This does not explain the anomaly. It defies common sense to believe that birds can distinguish between a loud noise and a predator, and then modify their behavior accordingly. (E) * This answer is correct; it explains the anomaly. When birds fly toward windowpanes, they are actually flying toward the reflection of the surrounding vegetation. To fly toward vegetation, or what the birds perceive to be vegetation, is consistent with their behavior in threatening situations.

Page 19: 1 1 1 1 1 1 - dl.keywin.orgdl.keywin.org/5/4/54bc444545de55e9a8148ce94764a168.pdf · 1 1 1 1 1 1 Analysis of the ... Why does the LSAT test your ability to evaluate an argument? Because

Get Prepped! PrepTest 28

19

4. Raising the humidity- This is a make a conclusion type question with a twist. So far, this section has been easy, but that will end soon. The correct answer is the only choice that is not supported by the passage. (A) * This answer is correct; it contradicts the passage. The passage explicitly stated that raising the humidity of a room protects computers from damage from excessively dry air. (B) This answer is incorrect; the passage supports it. The passage stated explicitly that humidity can be good for the furnishings of a room. (C) The passage stated that humid air feels warmer than dry air. (D) The passage stated that humid air could help alleviate some skin rashes. (E) This answer is incorrect; it is supported by the passage. The passage stated that humid air helps the body�s defense against viruses. 5. Jane- This is vulnerable to criticism question, which are very similar to flawed reasoning questions. It is the first of two that follow a script or dialogue type passage. Frequently, the first question addresses the interaction of the two speakers and the second addresses only the part of speaker number two. However, in this case, the first question addresses Maurice�s part, speaker number two. (A) This does not identify the weakness in Maurice�s argument. Jane� argument, which he refuted, is a proposal, not a policy. He also cited a fact to show that the proposal will fail. (B) This answer is incorrect; it is very convoluted and very confusing; however, words like �moral permissibility� should tip you off. (C) * This answer is correct; it accurately points out the weakness in Maurice�s argument. He mistakenly assumed that the cause of teenage violence has remained the same for hundreds of years. He failed to consider that teenage violence may be the same symptom but the diseases may be different. (D) This answer is incorrect; the fact cited is verifiable. (E) This answer is incorrect; there was no ambiguity in the term violence. 6. Jane- This is a strengthening question. The LSAT tests your ability to do this because it is an important skill to law students and lawyers. Being able to handle these questions efficiently is central to doing well on the test; so practice, practice, and practice them some more. To answer these questions correctly, you must understand the author�s conclusion. You want to be able to paraphrase it and then marshal evidence for it. This question asks you to strengthen the argument. (A) This answer is incorrect; it neither strengthens nor weakens the argument. Jane addressed only movies that depict violence among teenagers, not adults. (B) * This answer is correct; it tends to strengthen Jane�s argument by showing a connection between the cause of the problem and its solution. As a solution, Jane proposed prohibiting depictions of violence among teenagers in those movies promoted to young audiences. Given this new fact, Jane�s proposal is more likely to succeed because it addresses the most influential movies. (C) This answer is incorrect; it does not strengthen Jane�s argument. Profit is an irrelevant issue. (D) This answer tends to weaken Jane�s argument. It is evidence that adolescents, having never seen a violent movie, are still pre-disposed to violent behavior. (E) If they voluntarily restrict the subject matter to avoid violent themes, then a prohibition is less necessary. 7. Sam- This is an A responds to B question. How does Tiya�s response undercut Sam�s argument? Sam concluded that the cars are remarkably free of defects. (A) Tiya did not agree with Sam�s conclusion. (B) Tiya did not question whether the survey respondents real were truly satisfied. (C) * This answer is correct; Tiya did not question the results of the survey; however, she did question whether the survey was an adequate basis for Sam�s conclusion. (D) This is not how Tiya�s responded to Sam�s argument. (E) Close, but Tiya presented information that diminished the significance of Sam�s evidence, she did not imply that his conclusion was false. 8. Some environmentalists- This is a make conclusion question. It is pretty tough. Let�s make it easier by editing the facts. Some question exploitation on economic grounds. Many claim intrinsic value in spite of economic benefits. (A) This answer is incorrect; it is too certain of a conclusion to be based upon the passage. The passage stated that some environmentalists believe there is no economic benefit to exploiting the environment. It did offer concrete evidence that it is, in fact, economically imprudent. (B) * This answer is correct; it is the only choice that, based on the passage, is a certain conclusion. Some

Page 20: 1 1 1 1 1 1 - dl.keywin.orgdl.keywin.org/5/4/54bc444545de55e9a8148ce94764a168.pdf · 1 1 1 1 1 1 Analysis of the ... Why does the LSAT test your ability to evaluate an argument? Because

Get Prepped! PrepTest 28

20

environmentalists claim that the environment has intrinsic value (as opposed to economic value). (C) This answer is incorrect; it is too broad of a conclusion to be based upon the passage. The passage stated only that some environmentalists appeal to economic reasons. It is invalid to conclude that some means most. (D) This answer is incorrect; the passage did not address those environmentalists that appeal to only a non-economic justification. (E) This answer is incorrect; the passage only addressed what some environmentalists believe. It did not provide a basis to reach a conclusion about the justifications for protecting the environment. 9. Market research- This is a strengthen the argument question. The conclusion is that observation reveals more information than surveys. (A) * This answer tends to strengthen the conclusion that observational research provides information that surveys alone cannot. It allows researchers to gain information about those consumers who are unable to meaningfully participate in traditional surveys. (B) It would be a big stretch for this to be relevant. It fails to show a reason why observational surveys would add anything beyond traditional surveys. (C) This answer is irrelevant. (D) This answer is obviously incorrect; do not waste your time. (E) This answer is incorrect; the information is irrelevant to the researchers� conclusion. It does not address observational research. 10. Laura- This is an �A responds to B question.� Ralph says that Laura is missing the point because: (A) * This is the correct answer; it describes what Ralph pointed out in his response. To not be lonely, a person needs to be around other people. But, being around other people does not guarantee that a person will not be lonely. (B) This answer is incorrect; it does not describe what Ralph pointed out in his response. He agreed that to be around other people was needed to not be lonely. (C) This answer is incorrect. While true, this answer does not describe what Ralph pointed out in his response. (D) Ralph did not point out various other solutions. (E) Ralph did not point out that being around other people might make Harold even more lonely. 11. A rise in- This is a weakening question. The conclusion is that the 18 year-old recruitment rate is highly related to the 18 year-old dropout level. (A) This answer is incorrect; it is irrelevant information. The argument compared the rate, or percentage, of 18-year-olds, not the absolute number of 18-year-olds. (B) This answer is incorrect; it does not weaken the argument. Whether a high school education is necessary to operate certain equipment is actually irrelevant in determining the rate of 18-year-old high school dropouts who were recruited by the army. (C) This answer is correct; it weakens the argument. It provides evidence that, while the high school dropout rate increased, specifically among 18-year-olds it decreased. Because the army only recruited 18-year-olds, the percentage of recruits that were high school dropouts decreased. Therefore, the recruitment rates for 18-year-olds do not depend on the recruitment rates for high school dropouts. (D) This is irrelevant to the specific conclusion. (E) This answer is incorrect; it does not directly address the issue of whether the recruitment rate depends on the dropout rate. 12. Letter to the- This is a miscellaneous question type. The correct answer does not conflict with the letter writer�s facts. (A) This answer is incorrect; it conflicts with the writer. The writer stated that Vexone has been used effectively against all the species of cockroach that infest North America. (B) This answer conflicts with the writer�s view. The writer stated that studies prove that Roach Ender is effective against all species. (C) * This answer is correct; it agrees with the letter writer�s facts. (D) This answer is incorrect; it directly contradicts the writer�s assertion. (E) This answer is incorrect; the writer stated that Roach Ender was tested against all of the more than 4,000 species that infest N. America. 13. A recent study- This is an unusual resolve the paradox question. Birds either evolved from cold-blooded or warm-blooded ancestors. Look for a piece of evidence that tips the scales in favor to one side or the other (cold-blooded or warm-blooded). (A) This answer is incorrect; it does not help to resolve the dispute. It fails to address the issue; modern birds may be, or may not be, a warm-blooded species that descended from cold-blooded species.

Page 21: 1 1 1 1 1 1 - dl.keywin.orgdl.keywin.org/5/4/54bc444545de55e9a8148ce94764a168.pdf · 1 1 1 1 1 1 Analysis of the ... Why does the LSAT test your ability to evaluate an argument? Because

Get Prepped! PrepTest 28

21

(B) This answer is incorrect; it addresses factors from outside the passage (other physical traits of cold-blooded species) without shedding light on the dispute. (C) This answer is obviously incorrect; it is imbecilic. Whether or not modern birds evolved from pre-historic birds is not in dispute. Both sides agree, as should everyone, that modern birds evolved from prehistoric birds. (D) This answer is incorrect; it does help to resolve the dispute. Birds may be, or may not be, one of the warm-blooded species with dense blood vessels. (E) * This answer is correct; it helps to resolve the dispute in favor of the cold-blooded camp. It shows that having both growth rings and dense blood vessels is compatible with being cold blooded. 14. If citizens do- This is a make a conclusion question. Here the conclusion has largely been stated, you need only re-state it. Approach these as you would a main idea question from reading comprehension. They are straightforward and simple; do not make them difficult; do not infer anything beyond the passage; do not construe the conclusion more broadly than is necessary; and do not consider the validity or correctness of the conclusion. Finally, do not confuse the conclusion with either a fact or a premise from the passage. There is one exception to the above warning that stated that you should not infer anything beyond the passage. Sometimes there is an overlap between identify a conclusion and make a conclusion type questions. When this happens, the conclusion will be almost entirely stated within the passage, but you may have to connect the dots to find it. (A) * This answer identifies the conclusion the author had made. If you read the question before the passage, then you would have seen that there is no reason to read beyond the first sentence of the passage. The author stated the conclusion in the first sentence; so, everything afterwards is meaningless. (B) The theft example was an illustration, not necessarily an equal evil. (C) Imitation was not discussed. (D) This answer misstates the passage. A single person�s vote makes only an imperceptible difference. It is the widespread failure to vote that creates problems. (E) �Other societies� is too broad. It does not confine itself to the author�s stated main conclusion. 15. Human beings- This is a weaken the argument question. Which fact pours cold water on the conclusion that cognitive (intellectual) faculties must be gratified above all others? (A) The information is irrelevant to the argument, which deals only with human beings, not communication. (B) * This answer is correct; it tends to weaken the argument. It is evidence that, even if a human is aware of their superior intelligence, they can be made happy by something that does not involve gratification of their intellect. (C) Whatever. This one is out there. (D) This answer is incorrect; a serious athlete may be made happy by use of his or her cognitive faculties. (E) This answer is obviously incorrect. 16. Historian- This is an identify an assumption question. The correct answer is not explicitly stated within the passage; you must read between the lines. The passage contains the explicit conclusion that if they find mercury, the hypothesis is correct. Now work backwards to reveal the hidden assumption, the missing premise, or the implicit factual basis. The answer is a logical antecedent, or a necessary pre-condition, of the conclusion. Be leery of an answer choice that merely restates a fact from the passage or an answer choice that introduces a topic wholly unrelated to the passage and its conclusion. (A) This is not an assumption on which the historian�s argument depends. So long as a trace of mercury remains the conclusion is valid. It is irrelevant to the stated conclusion whether some or none of the mercury can be eliminated from the body. (B) * The author�s argument depends on this assumption. To plug this assumption into the argument is the best way to demonstrate why this answer is correct. Some people of Beethoven�s time did not ingest mercury (necessary assumption). Among the people who did ingest mercury were those who had a venereal disease. If Beethoven�s hair tests positive for traces of mercury, then he probably had a venereal disease (stated conclusion). Without this assumption, the historian�s argument is neither valid nor persuasive. Maybe everyone ingested mercury. (C) The efficacy of mercury to cure venereal diseases is not relevant to the stated conclusion. It is only important to the argument that persons with venereal diseases ingested mercury. (D) This cannot be an assumption on which the historian�s argument depends; it contradicts the historian�s argument. (E) These two issues are irrelevant.

Page 22: 1 1 1 1 1 1 - dl.keywin.orgdl.keywin.org/5/4/54bc444545de55e9a8148ce94764a168.pdf · 1 1 1 1 1 1 Analysis of the ... Why does the LSAT test your ability to evaluate an argument? Because

Get Prepped! PrepTest 28

22

17. In 1992- This is a weaken the argument question. The executive concludes that the lower salaries are justified by the experience they are gaining. (A) This answer strengthens the justification. (B) * This answer is correct; it undermines the justification. If most of the workers have been with the newspaper for more than ten years (and so are presumably experienced), then the average salary is the pay of veteran reporters, and not young reporters looking to gain valuable experience. (C) This answer is incorrect; it does not weaken the executive�s justification. The information is irrelevant. (D) This would explain why the salaries are low, but doesn�t weaken the executive�s justification. The information is not important to the executive�s justification. (E) This is wholly irrelevant information. 18. The human brain- This is a strengthen the argument question. (A) This answer merely restates part of the passage. (B) This answer introduces new and irrelevant information. (C) * This answer is correct; it tends to strengthen the argument. The passage provided a specific example of an argument that follows, or ascribes to, this general principle. (D) In the real world, this answer may be logical and true; but here, it fails to address or strengthen the argument. (E) The passage did not mention proliferation of a species as a factor to consider. 19. On a certain- This is an identify an assumption question. The conclusion is that the cancellations were not due to mechanical problems. (A) * The argument depends on this assumption. Working backwards from the stated conclusion, it is clear that this is the correct answer. Some flight delays were due to something other than mechanical problems (stated conclusion). There were nine flights that were delayed. Normally, only one or two airplanes experience delays because of mechanical problems. So, there must have been more than one or two planes that were scheduled for the nine canceled flights (necessary assumption). (B) This is irrelevant to the stated conclusion. (C) This is irrelevant to the stated conclusion. (D) This answer is incorrect; it is irrelevant to the stated conclusion. (E) This answer is incorrect; while it tends to support the stated conclusion, the conclusion does not depend on this assumption. 20. Game show host- This is a flaw in the reasoning question. The correct answer is the one that does not point out a flaw in the host�s reasoning. So we will see four flaws. (A) This answer is incorrect; it identifies a flaw in the game show host�s reasoning. Based on one chimp�s performance, the host concludes that all apes are capable of besting stock analysts. (B) * This answer is correct; it does not identify a flaw in the game show host�s reasoning. The host stated only that chimps are able to attain a better return, not that chimps are better able to understand investing. Whether chimps are capable of understanding stock reports is not relevant to the host�s conclusion. (C) This answer is incorrect; it identifies a flaw in the game show host�s reasoning. See (A). (D) Yes, a one-month experiment is not a great experiment. This answer is essentially identical to choice (E). Because two answers cannot both be correct, you know they are both wrong. (E) This answer is incorrect for the same reason as choice (D). 21. If the law- This is a parallel reasoning question. One of the ways the LSAT tests your logical ability is to ask you mirror a set of conditions. It is important to practice these because they are difficult and unfamiliar. For the uninitiated these can be time-consuming traps. Once you have practiced, you should be able to handle a parallel reasoning question in a minimal amount of time. On the exam, if you are pressed for time, it may be best to attempt to answer these questions last. It sometimes is easier to identify patterns of reasoning when an argument is in diagram form. Use words or symbols to diagram the argument. You should only be concerned with the blueprint, or design, of the argument. Beware of answer choices that deal with the same subject matter, or incorporate the same wording as the passage; generally, they are red herrings. The right answer is not necessarily identical to the passage, but it is the most similar in its reasoning. First, identify the flaw. Just because there is no law does not mean a city has no obligation to provide cans. Maybe public health reasons obligate a city to provide cans. Second, diagram or paraphrase the argument. If A (law punishes littering) then B (city has obligation). Since no A then no B. That was

Page 23: 1 1 1 1 1 1 - dl.keywin.orgdl.keywin.org/5/4/54bc444545de55e9a8148ce94764a168.pdf · 1 1 1 1 1 1 Analysis of the ... Why does the LSAT test your ability to evaluate an argument? Because

Get Prepped! PrepTest 28

23

easy enough. Our third step is to eliminate suspicious answer choices. Both (C) and (E) are suspiciously similar to the subject matter. Finally, diagram the remaining choices.

(A) If A (holiday) then B (open). Since no B then no A. This doesn�t mirror the argument. (B) If A (party) then B (balloons). Since no B, then no birthday. Totally wrong. (C) If A (adhere) then B (successful). Since A then B. No flaw here. (D) If A (late) then B (miss it). Since no A then no B. Here we go. Note that we had to do a

great deal of editing to find this. (E) If A (enforced) then B (jailed). Since no B then no A. Not the correct pattern.

22. Remember- This is an identify an assumption question. The facts are: people produce pherm. People voluntarily choose how to behave. Therefore, pherm don�t control behavior. (A) This answer is incorrect; the researcher�s argument did not depend on this assumption. The researcher�s argument depended on a distinction between voluntary and involuntary behavior, not on a distinction between chemical and psychological explanations. (B) * This answer is correct; it is an assumption on which the researcher�s argument depends. The researcher stated that the involuntary sexual behavior of animals has a purely chemical explanation. In contrast, human sexual behavior is purely psychological and, thus, voluntary (stated conclusion). So, voluntary action does not have a chemical explanation (necessary assumption). (C) This is true based on the passage, but it is not an assumption necessary to the stated conclusion. (D) �Evolution� is not an assumption on which the researcher�s argument depends. (E) This answer contradicts the passage. 23. Ethicist- This is an argument structure question. Understand how the argument plays out. The principle is that it is moral to lie, if the truth will cause mental or physical harm to others. (A) She lied to help him benefit, not to avoid harm. That is a big difference. (B) The father does not seek to avoid harming the daughter; he just couldn�t be bothered. (C) Harm, not convenience, is to be avoided. (D) * This answer is correct. The mother lies to avoid psychologically hurting the boy. (E) This is incorrect because he seeks to avoid causing pain to himself, not others. 24. Surviving seventeenth-century Dutch- This is a strengthening question. The conclusion is that there are too many paintings attributed to major painters to all be legitimate. What fact shows why there are too many major�s paintings? (A) This answer is incorrect; if true, it tends to weaken the argument. (B) This is neutral; it does not strengthen the argument. (C) This weakens the conclusion. (D) * If true, this shows why there are too many paintings attributable to majors. (E) This would be neutral, since it affects both equally. 25. The interstitial nucleus- This is a weaken the argument question. Weaken the conclusion that the size of the brain thingy causes the disease. The answer will focus on cause and effect. (A) This is irrelevant. (B) This is irrelevant. (C) This answer strengthens the argument. (D) This answer is incorrect; it does not weaken the argument. It is subtly phrased to confuse you. (E) * This is the correct answer; it weakens the conclusion. It is evidence that disease X is very similar to disease Y. If the size of the interstitial nucleus does not cause Y, then it probably doesn�t cause X. 26. It is common- This is a parallel reasoning question. It is not as involved as question number 21. To paraphrase this passage: It is common to refute the validity of an argument by showing that the person who is making the argument does not ascribe to it. This is irrational. Whether the person making the argument practices what she preaches, is irrelevant to the argument�s validity. Select the answer choice that proceeds in a similar fashion. (A) The first country did not exhort the other countries. (B) Seeing the benefits before doing the act is not part of the original argument. (C) Here, the second country doesn�t tell the first to stop. It uses the attack as a distraction. (D) * This one gets to the point. An argument can be valid, even if the one making the argument does not act as though the argument is true. (E) Condemning the few for the wrongs

Page 24: 1 1 1 1 1 1 - dl.keywin.orgdl.keywin.org/5/4/54bc444545de55e9a8148ce94764a168.pdf · 1 1 1 1 1 1 Analysis of the ... Why does the LSAT test your ability to evaluate an argument? Because

Get Prepped! PrepTest 28

24

of the many bears no resemblance to the passage�s last line.

Page 25: 1 1 1 1 1 1 - dl.keywin.orgdl.keywin.org/5/4/54bc444545de55e9a8148ce94764a168.pdf · 1 1 1 1 1 1 Analysis of the ... Why does the LSAT test your ability to evaluate an argument? Because

Get Prepped! PrepTest 28

25

(334-341)

4 4 4 4 4 4 General overview of Reading Comprehension-

There are several different overall strategies that you might use in this section. Without delving into too much detail, you may want to skim the questions before reading the passage. Or, you may want to use the special outlining techniques that help divide the passage into a usable format. Finally, some students find it helpful to skim the passage looking for the specific key words and issues. This is the one section where you will have to experiment with each of the major techniques to decide which works best given your personal reading style.

There are some strategies that everybody can use. First, you will find that here, like in the Logical Reasoning section, there are a limited number of question types, and they appear again and again (and again and again.) Learn to identify them so that you will instinctively read the passage looking for these question types. Learn to read for a fairly narrow set of issues. Second, it is a good idea to glance at all four passages before starting. You want to see if any passage deals with a subject matter with which you are familiar. Also, you want to see if any passage has significantly more questions associated with it. If you find either of these to be the case, then that passage is the best passage to start with, especially if you normally do not finish all four passages.

Overview of the entire section-

This section was more difficult than the average LSAT reading comprehension section. In all, there were twenty-six questions; ten details, seven inferences, five miscellaneous questions, and four main ideas. The first and final passages each had five questions, while the second and third had eight. The first passage, about some Native American tribes� attempt to establish land claims in court, was the second easiest. The second passage, about the climatic effects of volcanic eruptions, was the second most difficult. The third passage, about competing economic theories, was the most difficult of the passage. The fourth passage, about Hollywood�s relationship with the press, was the easiest. Overview of passage #1- Mashpee Wampanoag

This passage was more difficult than the average passage. Even so, it was the second easiest of the entire section. The subject matter covers the problems that can arise between cultures whose systems of discourse are different (as demonstrated by the Mashpee land claim suit). This is unfamiliar territory for most readers. The author presented the information in a straightforward manner and did not use many complex terms. The passage had only five questions; two main ideas, one inference, one detail, and one miscellaneous. 1. Main Idea- Main idea questions are the most commonly asked and easiest to answer. To answer a main idea question, you must identify the central, most important, message from the passage- the main idea, the primary purpose, the best title, the best description, or the best summary. Be careful, inevitably, at least one answer choice will accurately describe some aspect of the passage, or a portion of the main idea, and still not entirely capture the main point of the passage. With a main idea question, eliminate answers that focus only on specific information. Which of the following is a complete and accurate statement of the passage�s main idea? Remember, sometimes the main idea is summarized in the last sentence of the first paragraph. (A) This does not capture the passage�s main idea, and it also contradicts the passage. The passage stated that claims similar to the Mashpee�s have recently met with greater success (line 55). (B) This does not reflect the main idea. Although the legal definition of tribe was discussed, the main point was that the Mashpee�s suit underscored the need for US courts to accommodate differences in discourse between cultures (lines 14-15, 52-56). (C) * This answer is correct; it is an accurate and complete restatement to the passage�s main idea (see lines 12-15). (D) The court system never claimed to accept oral evidence, which it then denied. (E) This answer looks promising, but it is an attractive wrong answer. It accurately restates a portion of the passage, but it fails to capture the main point of the passage. It is a minor point. Plus, in

Page 26: 1 1 1 1 1 1 - dl.keywin.orgdl.keywin.org/5/4/54bc444545de55e9a8148ce94764a168.pdf · 1 1 1 1 1 1 Analysis of the ... Why does the LSAT test your ability to evaluate an argument? Because

Get Prepped! PrepTest 28

26

lines 52-55 the courts apparently disregard precedent in order to accommodate the different system. 2. Detail- Detail questions are also very common. The correct answer is expressly stated in the passage. It is waiting for you to dig it up. According to the passage, the Mashpee lawsuit was filed because they objected that the lands were sold, in violation of a statute that prohibits the transfer of tribal land without federal approval (lines 26-29). (A) This answer looks promising, but it is an attractive wrong answer. The passage stated that the non-Native American population increased in the town during the 1960's, but it was not this alone that caused the lawsuit. (B) The passage did not state that the statute was repealed. (C) This misstates the passage. After the town�s incorporation, The Mashpee Indians continued to have control over land use (see lines 19-21). (D) This is an obviously incorrect answer; it contradicts the passage. There was no written deed. (E) * This answer is correct; it accurately restates the relevant facts found in lines 24-29. 3. Miscellaneous/Attitude- Miscellaneous questions cover a wide number of topics. They may ask you about physical organization, attitude, or for an analogy. On an attitude question describe the author�s stance, beliefs, feelings, outlook, view, or opinion. If the author were giving a speech on this subject, what would she or he say, and how would he or she come across? While this question is certainly an attitude question, the quotation feature is unique. Select the quote that most clearly reveals the author�s attitude toward the court�s decision. (A) This quote refers to the court. (B), (C) & (D) These simply are stated facts. (E) * This answer is correct, the author felt the court failed to accommodate the cultural differences (lines 47-55). Note that �failure� is a more charged word than any of the other choices. 4. Inference- Inference questions appear about 20% of the time. Inference questions are typically the most difficult because, unlike a detail question, the answer is not expressly stated in the passage. These questions require you to make a sound conclusion based on the facts. Based on clues from the passage, which one of the following has happened since the lawsuit? An obvious place to look is in the part of the passage that comes after the lawsuit (lines 50-56). (A) This is an obviously incorrect answer; the passage stated explicitly that the Mashpee were unsuccessful in regaining control of the land. Therefore, it is very unlikely that, in the years since, they have regained control. (B) This is an obviously incorrect; the passage stated that recent claims have met with greater success, but there is no reason to infer that, in the years since, tribes have won all (red-flag word) their land claims in court. (C) This is an incorrect answer; it lacks support in the passage. There was nothing in the passage to indicate that U.S. courts no longer abide by the statute. (D) * This is the most reasonable inference. The passage stated that �U.S. courts have begun to acknowledge the failure to accommodate differences...�; It is reasonable to assume that courts are more likely to accept oral testimony. 5. Main Idea- The passage is primarily concerned with which of the following? Often the first and last questions are main idea, big picture, questions. (A) This is an incorrect answer; the passage did not evaluate various approaches to solving a problem. (B) * This answer is correct; it reflects the passage�s primary concern. The passage illuminated the general problem between cultures whose systems of discourse differ, by discussing a specific example, the Mashpee lawsuit. (C) The passage was primarily concerned with how the court system approached the problem; there was no opposing side. (D) The passage did not criticize an earlier solution to the problem, nor did it mention new information. (E) This is an incorrect answer; the passage did not include an earlier analysis, nor propose a new solution.

Page 27: 1 1 1 1 1 1 - dl.keywin.orgdl.keywin.org/5/4/54bc444545de55e9a8148ce94764a168.pdf · 1 1 1 1 1 1 Analysis of the ... Why does the LSAT test your ability to evaluate an argument? Because

Get Prepped! PrepTest 28

27

Overview of passage #2- Climatic effects of volcanic eruptions This passage was more difficult than the average LSAT passage and the second most difficult of

the entire section. The subject, the effects of volcanic eruptions on the atmosphere, is unfamiliar territory for most people. The author presented the information clearly, but he or she used some complex terms, like �climatic feedback loop.� The passage had eight questions. That is a ton of questions. 6. Main idea- Avoid answers that only cover one element of the passage. (A) This is an obviously incorrect answer; it misstates a fact from the passage. The passage stated that volcanoes cause a smaller drop than expected (lines 34-35). (B) This is contradicted by line 35. (C) This is incorrect for the same reason as (A). (D) * This answer is correct. If you review line 40 you see the discussion of �regional fluctuations.� (E) This is interesting. The passage did mention that cyclical weather phenomenon affected calculations, but it was not the main idea of the passage since the passage went on to discuss the feedback loops hypothesis. 7. Detail- Be careful, this question is very difficult. Hopefully you skipped it if it took too long. The specific information that is relevant to this question was stated in the third paragraph, �Once El Niño effects had been subtracted from the data, the actual effects of the eruptions came through more clearly� (lines 13-15). The effects of El Niño are a factor that directly influences the average global temperature. (A) This is an incorrect answer; the situation is not similar to the specific example. The total weight of a package does not influence the weight of its contents, apart from the packing material. (B) This is an incorrect answer; the value of coins does not influence the number of coins in a pile. (C) This is an obviously incorrect answer; the magnification of a lens does not influence the shape of an object seen through the lens. (D) This is not similar to the passage. The number of false crimes reported does not directly influence the number of crimes actually committed in the city. (E) * This answer is correct; it is an analogous situation. The ages of new immigrants are one factor that directly influences the change in the average age of the country�s population. 8. Detail- Which of the following is not an effect of El Niño? This is a detail question with a twist. Normally the correct answer to a detail question is explicitly stated in the passage. In this instance, the correct answer is the only one that is not explicitly stated in the passage. (A) & (B) This is incorrect; it was explicitly stated in the passage (lines 23-24). (C) This was explicitly stated in lines 20-22. (D) * This is the correct answer; the passage did not explicitly mention this as an effect of El Niño. El Niño doesn�t cause feedback loops. (E) This is an incorrect answer; the passage indicated that this is an effect of El Niño (lines 27-29). 9. Detail- Which of the following most accurately reflects what the author meant by �minor� volcanic eruption? The author contrasted minor eruptions with major eruptions. The author described major eruptions as �dust-spitting explosions� (line 32); therefore, minor eruptions are not dust spitting explosions. (A) This is an incorrect answer; the size of an eruption is related to the amount of dust it produces, not the flow of lava. (B) This answer looks promising, but it is an attractive wrong answer. A minor eruption would, in fact, have no effect on global temperature, but that is not what differentiates a minor eruption from a major eruption. (C) This is an incorrect answer; the passage stated that minor eruptions had no effect on temperature. (D) * This answer is correct; a volcano with a small amount of debris most accurately characterizes what the author meant by �minor.� (E) This is an incorrect answer; the passage stated that minor eruptions had no effect on temperature. 10. Detail- The key to this question is the specific example from the passage�s last paragraph of a feedback loop (lines 41-56). You can think of the concept as: an initial increase in X causes Y; Y then causes Z; Z then causes another increase in X. (A) * This answer is correct; it accurately reflects the concept of a feedback loop as exemplified by the passage; X causes B, which causes C, which causes more of X. (B) This is an incorrect answer; it mischaracterizes the concept of a feedback loop. In this example,

Page 28: 1 1 1 1 1 1 - dl.keywin.orgdl.keywin.org/5/4/54bc444545de55e9a8148ce94764a168.pdf · 1 1 1 1 1 1 Analysis of the ... Why does the LSAT test your ability to evaluate an argument? Because

Get Prepped! PrepTest 28

28

the wolf population did not subsequently increase. (C) This is an incorrect answer; it mischaracterizes the concept of a feedback loop. In this example, the initial rain in an area did not cause a further increase in the subsequent amount of rain. (D) This is an incorrect answer; the initial increase in sunlight did not cause a subsequent increase in the amount of sunlight. (E) This is obviously not an example of a feedback loop. 11. Inference- The author of the passage is most likely to agree with which? (A) This is an incorrect answer; because the author specifically contradicted this statement (lines 35-36), you cannot reasonably assume that the author would agree. (B) This is an incorrect answer; the passage stated nothing about the effect of volcanic eruptions upon El Niño. (C) * This answer is correct, based on the passage, it is likely that the author would agree with this statement. We need a feedback loop to cause the cold weather. (D) This is an incorrect answer, minor eruptions do not have less of an effect, they have no effect at all. (E) This is an incorrect answer because the author specifically contradicted this (lines 20-22). 12. Inference- This question is awkwardly worded to confuse you, but it is an inference question. The only difference is that, in this instance, the correct choice is the one which is not a reasonable assumption based on information from the passage. (A) The information from the passage does support this claim. Major eruptions cause a smaller drop than expected, but it is discernable (lines 30-34). (B) This is an incorrect answer, for the same reason as (A). (C) * This answer is correct; information from the passage least supports this answer. The passage stated that major eruptions have a discernable effect on global temperature. Therefore (we infer), it seems very unlikely that a major eruption would have no discernable effect on regional temperature. (D) Information from the passage tends to support this statement (lines 30-31). (E) This is incorrect for the same reason as (D). 13. Miscellaneous/Organization- This question is very similar to a main idea question; however, it is only concerned with a single paragraph, not the entire passage. In this instance, you must identify the primary purpose of the last paragraph and keep in mind how it fit into the overall scheme. Recall, the last paragraph dealt exclusively with the concept of a feedback loop. (A) This is an incorrect answer; the year without a summer was the same as other examples of climatic feedback loops. (B) This is an incorrect answer; a climatic feedback loop magnifies the effect of a major volcanic eruption. (C) * This answer is correct; it captures the main idea of the final paragraph and reflects how it fit into the passage as a whole. (D) This is an incorrect answer; the final paragraph had nothing to do with El Niño. Note, this answer choice is the only one that mentions El Niño. It looks different than the others. When an answer choice sticks out because it is different, you can eliminate it outright. (E) The paragraph did not suggest that we modify the current model.

Page 29: 1 1 1 1 1 1 - dl.keywin.orgdl.keywin.org/5/4/54bc444545de55e9a8148ce94764a168.pdf · 1 1 1 1 1 1 Analysis of the ... Why does the LSAT test your ability to evaluate an argument? Because

Get Prepped! PrepTest 28

29

Overview of passage #3- Steady-state economics This passage was more difficult than the average LSAT passage and the most difficult of the entire

section. The subject, a debate between the neo-classical economic theory and the steady-state economic theory, was probably completely alien for you, unless you were an economics major. The author presented the information in a straightforward manner, but he or she used some specialized terms. The passage had a full eight questions; three inferences, two details, two miscellaneous questions, and one main idea. 14. Main Idea Question- This was a monster of a question. The answers were long, and several of them were attractive. (A) This answer looks promising, but it is an attractive wrong answer. It accurately and completely captures only one-half of the passage�s main idea, the neoclassical side. (B) This is an incorrect answer; it accurately restates elements from the passage, but fails to capture all the elements. (C) This is an incorrect answer; it restates only part of the passage�s main idea, the steady-state side. (D) * This answer is correct; it accurately and completely expresses the main idea of the passage. It correctly restates the neo-classical and the steady-state view and discusses human wants. (E) This is an obviously incorrect answer; it misstates a crucial fact from the passage. Steady-state economists do not view the economy as circular and do not want unimpeded growth in Western economies. 15. Inference Question- Based on the passage, it is likely that neoclassical economists would disagree with steady-state economists on each of the following except: (A) * This answer is correct; neoclassical economists believe that when natural resources run out, human-made resources will allow continued unlimited growth. Thus, it is safe to assume that neoclassical economists recognize nature�s limit to yield resources. (B) This is an incorrect answer; neoclassical economists do not view natural resources as an external constraint (lines 30-32). (C) This is an incorrect answer; neoclassical economists see unlimited growth as necessary and good (see lines 9-14). (D) This is an incorrect answer; neoclassic economists believe that growth is essential, so there is no optimal size to an economy; it is always expanding. (E) This is incorrect for the same reason as (D). 16. Detail- Steady-state economists believe that unlimited growth is dangerous because: (A) This is an incorrect answer, steady-state economists are interested in conservation (qualitative change), not further exploration (quantitative change). (B) This answer looks promising, but it is an attractive wrong answer. The statement is likely true, but it does not restate explicit information from the passage. (C) Generating new markets is a concern of neoclassical economists, not steady-state economists. (D) This is an incorrect answer; only neoclassicists view capital as necessary to redress income inequities. (E) * This answer is correct; it restates information from lines 22-26. 17. Inference- Which of the following, based on the passage, would a steady-state economist least likely agree with? Four choices will be agreeable, one will be disagreeable or neutral. (A) This is an incorrect answer; a steady-state economist would likely agree with a plan to recycle. (B) * This answer is correct; it is unlikely that a steady-state economist would endorse use of cheaper fuel, (it would likely increase consumption), as a means to reduce growth without compromising human wants. (C) This process, if implemented, would reduce growth and waste without compromising to consumer wants. (D) A steady-state economist would likely agree with this conservation measure. (E) A steady state economist would likely agree with a reduction in waste. 18. Inference- A steady-state economist is likely to agree with which one of the following principles? (A) This is an incorrect answer; this is the neoclassical view. (B) This is an incorrect answer; There is no basis from the passage to conclude whether a steady-state economist is likely or unlikely to agree with this principle. (C) * This answer is correct, information in the passage suggested that a steady-state economist was likely to agree with this principle (lines 30 and 56-59). (D) This is an obviously incorrect answer; if you know anything at all about economic theory, and even if you don�t, you know to reject this answer outright. (E) This is an incorrect answer; it is likely that only a neoclassical economist would agree with

Page 30: 1 1 1 1 1 1 - dl.keywin.orgdl.keywin.org/5/4/54bc444545de55e9a8148ce94764a168.pdf · 1 1 1 1 1 1 Analysis of the ... Why does the LSAT test your ability to evaluate an argument? Because

Get Prepped! PrepTest 28

30

this statement. 19. Detail- From the steady-state view, which of the following is a non-economic limit referred to in line 7? (A) This is an incorrect answer; the total amount of human wants fits within the neoclassical closed-system model. (B) This is an incorrect answer; an index is a measurement of economic activity, it does not affect the economy. (C) * This answer is correct; according to the passage, steady-state economists consider �nature�s limited capacity to... absorb waste,� an outside constraint on growth (line 22). (D) This is an incorrect answer; it does not fit within the definition of a non-economic constraint. (E) This is an incorrect answer; stagnation is an economic constraint. 20. Miscellaneous/Organization- Physical organization questions are concerned with the mechanics or the blueprint of a passage. Marking the passage�s main ideas, important arguments and transition terms provides clues to the structure of the passage; which is by far, the most important thing for you to recognize. Referring to your notes will help you sort-out how the passage was designed. What was the last paragraph about, and how did it fit into the overall scheme of the passage? This was a hard one. (A) This is an incorrect answer; the paragraph did not contradict the ways in which two schools interpret data. (B) This is an incorrect answer; the last paragraph did not recommend a policy based on the prevailing economic school. (C) This is an incorrect answer; the last paragraph did not argue against steady-state economics. It proposed a way to test it. (D) * This answer is correct; it correctly reflects the purpose of the last paragraph. Implementing policies to limit growth draws an objection from neoclassicists that this will cause stagnation. But a policy of conservation may satisfy wants. (E) This is an incorrect answer; the two schools agree on the basic goal of an economy � satisfying wants. 21. Inference- The passage suggested which about neo-classical economists? (A) This is an incorrect answer; neoclassical economists assume that human-made resources are infinitely available to replace depleted natural resources and sustain unlimited growth. (B) * This answer is correct; for the same reason as (A). (C) This is an incorrect answer; it reflects the views of steady-state economists. (D) This is an incorrect answer; there was nothing in the passage that suggested this. (E) This is an incorrect answer; there was nothing in the passage to suggest this.

Page 31: 1 1 1 1 1 1 - dl.keywin.orgdl.keywin.org/5/4/54bc444545de55e9a8148ce94764a168.pdf · 1 1 1 1 1 1 Analysis of the ... Why does the LSAT test your ability to evaluate an argument? Because

Get Prepped! PrepTest 28

31

Overview of passage #4- Hollywood and the press This passage was as difficult as the average LSAT passage and the easiest of the entire section.

The subject, the interdependent mass-media entertainment industry, is familiar territory for most people. The author presented the information in a straightforward manner, and used simple terms. The passage had five questions; three details, one inference, one miscellaneous, and no main idea, which is very unusual. 22. Inference- The author would be most likely to agree with which conclusion: (A) * The author pegs the mid-1920s as the start of this phenomenon, which made movies less engaging. Thus, the movies used to be more engaging. (B) Conclusions about the relative talents lacks a factual foundation. (C) The passage suggested the opposite, that studios, today, are interdependent with mass media. (D) The passage suggested the opposite. Today, profit comes from exciting an audience�s curiosity about a movie; therefore, the publicity is probably interesting. (E) This is an incorrect answer; the difference is attributable to an increased emphasis on publicity away from movie making. 23. Detail- According to the author, �the new danger is that profit comes only from satisfying an audience�s curiosity...� (lines 43-47). (A) This is an incorrect answer; the mass media encourages such filmmakers. (B) This is an incorrect answer; it may or may not be true, but it was not expressly stated in the passage. (C) * This answer is correct; it is an accurate restatement of lines 43-47. (D) This is an incorrect answer; it may or may not be true, but it was not expressly stated. (E) This is an incorrect answer; it may or may not be true, but there was no indication that audiences are cynical. In fact, it would seem that they are very easily duped into seeing whatever they are told to. 24. Detail- The phrase �cultural cross-fertilization� appeared in line 17. (A) This is an incorrect answer; the term did not refer to a competitive relationship. (B) This is an incorrect answer; the relationship was between movies and the press. (C) This is an incorrect answer; it fails to account for the mutually beneficial aspect of the relationship. (D) * This answer is correct; it accurately reflects the definition ascribed to �cultural cross-fertilization.� (E) This is an incorrect answer; the relationship was between film studios and the press, not public relations agents and the press. 25. Miscellaneous/Organization- Which of the following best describes the layout of the passage? Note that the first three choices start with the same language, so you can ignore at least that aspect of these choices. (A) This is an incorrect answer; the passage did not conclude with an introduction of a possible solution to the problem. (B) * This answer is correct; it accurately describes the overall organization of the passage. (C) This is an incorrect answer, the passage did not introduce a possible solution to the problem. (D) This is an incorrect answer; the passage did not appraise us of the importance of avoiding this consequence. (E) This is an incorrect answer; the passage did not appeal for assistance to solve the problem. 26. Detail- Which one of the following, if it were true, would most weaken the author�s position in lines 35-47. (A) This is an incorrect answer; it does not weaken the author�s position, the danger is posed by the desire to make a profit. (B) This is an incorrect answer; it does not account for the danger posed by the desire to make a profit. (C) This is an incorrect answer for the same reason as choice (B). (D) * This answer is correct; if it were true, the desire to make a profit would not jeopardize the ability of a movie to satisfy movie-goers emotionally. (E) This is an incorrect answer; if it were true, it would strengthen the author�s position.